Max and min of $f(x,y)=e^x+e^y$ on $x^2+y^2=1$












0












$begingroup$


$V={(x,y):x^2+y^2=1}$ is compact and f is continuous on V so for Weierstrass global max and min exist.To find them I have used lagrange multipliers but the system don't give a clear solution.










share|cite|improve this question









$endgroup$












  • $begingroup$
    I have a wild shot... Substitute $y$ by $pmsqrt{1-x^2}$ and put it in $f$. Then apply normal differentiation process to find maxima and minima. I don't know what this will result, just a vague idea to tackle such problems.
    $endgroup$
    – Anik Bhowmick
    Jan 31 at 9:42












  • $begingroup$
    One can find the minima in a completely elementary manner - by AM-GM, $e^x + e^y ge 2e^{(x+y)/2}$, with equality along the line $x = y.$ But trivially the minimiser of $(x+y)$ on the unit circle also lies on this line, and immediately we have that the minima of the original objective is $2exp(-1/sqrt{2}).$ Question: can the maxima be arrived at through some similar elementary method? I can't quite think of one.
    $endgroup$
    – stochasticboy321
    Jan 31 at 9:58
















0












$begingroup$


$V={(x,y):x^2+y^2=1}$ is compact and f is continuous on V so for Weierstrass global max and min exist.To find them I have used lagrange multipliers but the system don't give a clear solution.










share|cite|improve this question









$endgroup$












  • $begingroup$
    I have a wild shot... Substitute $y$ by $pmsqrt{1-x^2}$ and put it in $f$. Then apply normal differentiation process to find maxima and minima. I don't know what this will result, just a vague idea to tackle such problems.
    $endgroup$
    – Anik Bhowmick
    Jan 31 at 9:42












  • $begingroup$
    One can find the minima in a completely elementary manner - by AM-GM, $e^x + e^y ge 2e^{(x+y)/2}$, with equality along the line $x = y.$ But trivially the minimiser of $(x+y)$ on the unit circle also lies on this line, and immediately we have that the minima of the original objective is $2exp(-1/sqrt{2}).$ Question: can the maxima be arrived at through some similar elementary method? I can't quite think of one.
    $endgroup$
    – stochasticboy321
    Jan 31 at 9:58














0












0








0


2



$begingroup$


$V={(x,y):x^2+y^2=1}$ is compact and f is continuous on V so for Weierstrass global max and min exist.To find them I have used lagrange multipliers but the system don't give a clear solution.










share|cite|improve this question









$endgroup$




$V={(x,y):x^2+y^2=1}$ is compact and f is continuous on V so for Weierstrass global max and min exist.To find them I have used lagrange multipliers but the system don't give a clear solution.







real-analysis






share|cite|improve this question













share|cite|improve this question











share|cite|improve this question




share|cite|improve this question










asked Jan 31 at 9:32









user495707user495707

25




25












  • $begingroup$
    I have a wild shot... Substitute $y$ by $pmsqrt{1-x^2}$ and put it in $f$. Then apply normal differentiation process to find maxima and minima. I don't know what this will result, just a vague idea to tackle such problems.
    $endgroup$
    – Anik Bhowmick
    Jan 31 at 9:42












  • $begingroup$
    One can find the minima in a completely elementary manner - by AM-GM, $e^x + e^y ge 2e^{(x+y)/2}$, with equality along the line $x = y.$ But trivially the minimiser of $(x+y)$ on the unit circle also lies on this line, and immediately we have that the minima of the original objective is $2exp(-1/sqrt{2}).$ Question: can the maxima be arrived at through some similar elementary method? I can't quite think of one.
    $endgroup$
    – stochasticboy321
    Jan 31 at 9:58


















  • $begingroup$
    I have a wild shot... Substitute $y$ by $pmsqrt{1-x^2}$ and put it in $f$. Then apply normal differentiation process to find maxima and minima. I don't know what this will result, just a vague idea to tackle such problems.
    $endgroup$
    – Anik Bhowmick
    Jan 31 at 9:42












  • $begingroup$
    One can find the minima in a completely elementary manner - by AM-GM, $e^x + e^y ge 2e^{(x+y)/2}$, with equality along the line $x = y.$ But trivially the minimiser of $(x+y)$ on the unit circle also lies on this line, and immediately we have that the minima of the original objective is $2exp(-1/sqrt{2}).$ Question: can the maxima be arrived at through some similar elementary method? I can't quite think of one.
    $endgroup$
    – stochasticboy321
    Jan 31 at 9:58
















$begingroup$
I have a wild shot... Substitute $y$ by $pmsqrt{1-x^2}$ and put it in $f$. Then apply normal differentiation process to find maxima and minima. I don't know what this will result, just a vague idea to tackle such problems.
$endgroup$
– Anik Bhowmick
Jan 31 at 9:42






$begingroup$
I have a wild shot... Substitute $y$ by $pmsqrt{1-x^2}$ and put it in $f$. Then apply normal differentiation process to find maxima and minima. I don't know what this will result, just a vague idea to tackle such problems.
$endgroup$
– Anik Bhowmick
Jan 31 at 9:42














$begingroup$
One can find the minima in a completely elementary manner - by AM-GM, $e^x + e^y ge 2e^{(x+y)/2}$, with equality along the line $x = y.$ But trivially the minimiser of $(x+y)$ on the unit circle also lies on this line, and immediately we have that the minima of the original objective is $2exp(-1/sqrt{2}).$ Question: can the maxima be arrived at through some similar elementary method? I can't quite think of one.
$endgroup$
– stochasticboy321
Jan 31 at 9:58




$begingroup$
One can find the minima in a completely elementary manner - by AM-GM, $e^x + e^y ge 2e^{(x+y)/2}$, with equality along the line $x = y.$ But trivially the minimiser of $(x+y)$ on the unit circle also lies on this line, and immediately we have that the minima of the original objective is $2exp(-1/sqrt{2}).$ Question: can the maxima be arrived at through some similar elementary method? I can't quite think of one.
$endgroup$
– stochasticboy321
Jan 31 at 9:58










3 Answers
3






active

oldest

votes


















1












$begingroup$

The contour curves $y=ln (z-e^x)$ and the constraint circle $x^2+y^2=1$ are shown on the graph:



$hspace{1cm}$enter image description here



The min/max of $z(x,y)$ will occur when $x=y$.






share|cite|improve this answer









$endgroup$





















    1












    $begingroup$

    To find the maximum it is enough to consider positive values of $x$ and $y$ because $(x,y) in V$ implies $(pm x ,pm y) in V$. The method of Lagrange multipliers leads to the equation $xe^{y}=ye^{x}$. Note that the derivative of $frac {e^{x}} x$ is negative on $(0,1)$ so we get $x=y$. Hence the maximum is attained at $x=y=frac 1 {sqrt 2}$. Hint for minimum: the minimum value will be attained when $x,y<0$.






    share|cite|improve this answer











    $endgroup$





















      1












      $begingroup$

      No need to use Lagrange at all. Substitute $x$ and $y$ by $sin(t)$ and $cos(t)$ the problem becomes maximizing $g(t):=( e^{sin(t)} + e^{cos(t)} )$. Then $g'(t) = e^{sin(t)}cos(t) - e^{cos(t)} sin(t) $. Then $g'(t)$ then is zero iff
      $$ frac{cos(t)}{ e^{cos(t)}} = frac{sin(t)}{ e^{sin(t)}} $$
      Let $ h(t) = t e^{-t} $. Then the upper equation becomes $h(cos(t)) = h(sin(t))$ . As $ h $ is monotonic increasing(as $h'ge 0$ on [0,1]) the equation will only be true where $cos(t) = sin(t)$. So the equation will only be true for $t = pi/4, text{ or } 5pi/4 $, or $sin(t) =cos(t) = pm sqrt{2}/2 $. So Max will be attained as $2e^{sqrt{2}/2}$ and min as $2e^{-sqrt{2}/2}$






      share|cite|improve this answer











      $endgroup$









      • 1




        $begingroup$
        It is not true for $t = - pi/4 $. $sin(- pi/4)={- sqrt{2}/2}$ and $cos(- pi/4)={sqrt{2}/2}$
        $endgroup$
        – Piotr Wasilewicz
        Feb 1 at 10:02








      • 1




        $begingroup$
        Good catch! Corrected.
        $endgroup$
        – Maksim
        Feb 1 at 10:30






      • 1




        $begingroup$
        I think you mean $5pi/4$ :)
        $endgroup$
        – Piotr Wasilewicz
        Feb 1 at 11:21










      • $begingroup$
        You are so right :-)
        $endgroup$
        – Maksim
        Feb 1 at 11:53












      Your Answer





      StackExchange.ifUsing("editor", function () {
      return StackExchange.using("mathjaxEditing", function () {
      StackExchange.MarkdownEditor.creationCallbacks.add(function (editor, postfix) {
      StackExchange.mathjaxEditing.prepareWmdForMathJax(editor, postfix, [["$", "$"], ["\\(","\\)"]]);
      });
      });
      }, "mathjax-editing");

      StackExchange.ready(function() {
      var channelOptions = {
      tags: "".split(" "),
      id: "69"
      };
      initTagRenderer("".split(" "), "".split(" "), channelOptions);

      StackExchange.using("externalEditor", function() {
      // Have to fire editor after snippets, if snippets enabled
      if (StackExchange.settings.snippets.snippetsEnabled) {
      StackExchange.using("snippets", function() {
      createEditor();
      });
      }
      else {
      createEditor();
      }
      });

      function createEditor() {
      StackExchange.prepareEditor({
      heartbeatType: 'answer',
      autoActivateHeartbeat: false,
      convertImagesToLinks: true,
      noModals: true,
      showLowRepImageUploadWarning: true,
      reputationToPostImages: 10,
      bindNavPrevention: true,
      postfix: "",
      imageUploader: {
      brandingHtml: "Powered by u003ca class="icon-imgur-white" href="https://imgur.com/"u003eu003c/au003e",
      contentPolicyHtml: "User contributions licensed under u003ca href="https://creativecommons.org/licenses/by-sa/3.0/"u003ecc by-sa 3.0 with attribution requiredu003c/au003e u003ca href="https://stackoverflow.com/legal/content-policy"u003e(content policy)u003c/au003e",
      allowUrls: true
      },
      noCode: true, onDemand: true,
      discardSelector: ".discard-answer"
      ,immediatelyShowMarkdownHelp:true
      });


      }
      });














      draft saved

      draft discarded


















      StackExchange.ready(
      function () {
      StackExchange.openid.initPostLogin('.new-post-login', 'https%3a%2f%2fmath.stackexchange.com%2fquestions%2f3094682%2fmax-and-min-of-fx-y-exey-on-x2y2-1%23new-answer', 'question_page');
      }
      );

      Post as a guest















      Required, but never shown

























      3 Answers
      3






      active

      oldest

      votes








      3 Answers
      3






      active

      oldest

      votes









      active

      oldest

      votes






      active

      oldest

      votes









      1












      $begingroup$

      The contour curves $y=ln (z-e^x)$ and the constraint circle $x^2+y^2=1$ are shown on the graph:



      $hspace{1cm}$enter image description here



      The min/max of $z(x,y)$ will occur when $x=y$.






      share|cite|improve this answer









      $endgroup$


















        1












        $begingroup$

        The contour curves $y=ln (z-e^x)$ and the constraint circle $x^2+y^2=1$ are shown on the graph:



        $hspace{1cm}$enter image description here



        The min/max of $z(x,y)$ will occur when $x=y$.






        share|cite|improve this answer









        $endgroup$
















          1












          1








          1





          $begingroup$

          The contour curves $y=ln (z-e^x)$ and the constraint circle $x^2+y^2=1$ are shown on the graph:



          $hspace{1cm}$enter image description here



          The min/max of $z(x,y)$ will occur when $x=y$.






          share|cite|improve this answer









          $endgroup$



          The contour curves $y=ln (z-e^x)$ and the constraint circle $x^2+y^2=1$ are shown on the graph:



          $hspace{1cm}$enter image description here



          The min/max of $z(x,y)$ will occur when $x=y$.







          share|cite|improve this answer












          share|cite|improve this answer



          share|cite|improve this answer










          answered Jan 31 at 9:48









          farruhotafarruhota

          21.8k2842




          21.8k2842























              1












              $begingroup$

              To find the maximum it is enough to consider positive values of $x$ and $y$ because $(x,y) in V$ implies $(pm x ,pm y) in V$. The method of Lagrange multipliers leads to the equation $xe^{y}=ye^{x}$. Note that the derivative of $frac {e^{x}} x$ is negative on $(0,1)$ so we get $x=y$. Hence the maximum is attained at $x=y=frac 1 {sqrt 2}$. Hint for minimum: the minimum value will be attained when $x,y<0$.






              share|cite|improve this answer











              $endgroup$


















                1












                $begingroup$

                To find the maximum it is enough to consider positive values of $x$ and $y$ because $(x,y) in V$ implies $(pm x ,pm y) in V$. The method of Lagrange multipliers leads to the equation $xe^{y}=ye^{x}$. Note that the derivative of $frac {e^{x}} x$ is negative on $(0,1)$ so we get $x=y$. Hence the maximum is attained at $x=y=frac 1 {sqrt 2}$. Hint for minimum: the minimum value will be attained when $x,y<0$.






                share|cite|improve this answer











                $endgroup$
















                  1












                  1








                  1





                  $begingroup$

                  To find the maximum it is enough to consider positive values of $x$ and $y$ because $(x,y) in V$ implies $(pm x ,pm y) in V$. The method of Lagrange multipliers leads to the equation $xe^{y}=ye^{x}$. Note that the derivative of $frac {e^{x}} x$ is negative on $(0,1)$ so we get $x=y$. Hence the maximum is attained at $x=y=frac 1 {sqrt 2}$. Hint for minimum: the minimum value will be attained when $x,y<0$.






                  share|cite|improve this answer











                  $endgroup$



                  To find the maximum it is enough to consider positive values of $x$ and $y$ because $(x,y) in V$ implies $(pm x ,pm y) in V$. The method of Lagrange multipliers leads to the equation $xe^{y}=ye^{x}$. Note that the derivative of $frac {e^{x}} x$ is negative on $(0,1)$ so we get $x=y$. Hence the maximum is attained at $x=y=frac 1 {sqrt 2}$. Hint for minimum: the minimum value will be attained when $x,y<0$.







                  share|cite|improve this answer














                  share|cite|improve this answer



                  share|cite|improve this answer








                  edited Feb 1 at 11:41

























                  answered Jan 31 at 9:44









                  Kavi Rama MurthyKavi Rama Murthy

                  72.6k53170




                  72.6k53170























                      1












                      $begingroup$

                      No need to use Lagrange at all. Substitute $x$ and $y$ by $sin(t)$ and $cos(t)$ the problem becomes maximizing $g(t):=( e^{sin(t)} + e^{cos(t)} )$. Then $g'(t) = e^{sin(t)}cos(t) - e^{cos(t)} sin(t) $. Then $g'(t)$ then is zero iff
                      $$ frac{cos(t)}{ e^{cos(t)}} = frac{sin(t)}{ e^{sin(t)}} $$
                      Let $ h(t) = t e^{-t} $. Then the upper equation becomes $h(cos(t)) = h(sin(t))$ . As $ h $ is monotonic increasing(as $h'ge 0$ on [0,1]) the equation will only be true where $cos(t) = sin(t)$. So the equation will only be true for $t = pi/4, text{ or } 5pi/4 $, or $sin(t) =cos(t) = pm sqrt{2}/2 $. So Max will be attained as $2e^{sqrt{2}/2}$ and min as $2e^{-sqrt{2}/2}$






                      share|cite|improve this answer











                      $endgroup$









                      • 1




                        $begingroup$
                        It is not true for $t = - pi/4 $. $sin(- pi/4)={- sqrt{2}/2}$ and $cos(- pi/4)={sqrt{2}/2}$
                        $endgroup$
                        – Piotr Wasilewicz
                        Feb 1 at 10:02








                      • 1




                        $begingroup$
                        Good catch! Corrected.
                        $endgroup$
                        – Maksim
                        Feb 1 at 10:30






                      • 1




                        $begingroup$
                        I think you mean $5pi/4$ :)
                        $endgroup$
                        – Piotr Wasilewicz
                        Feb 1 at 11:21










                      • $begingroup$
                        You are so right :-)
                        $endgroup$
                        – Maksim
                        Feb 1 at 11:53
















                      1












                      $begingroup$

                      No need to use Lagrange at all. Substitute $x$ and $y$ by $sin(t)$ and $cos(t)$ the problem becomes maximizing $g(t):=( e^{sin(t)} + e^{cos(t)} )$. Then $g'(t) = e^{sin(t)}cos(t) - e^{cos(t)} sin(t) $. Then $g'(t)$ then is zero iff
                      $$ frac{cos(t)}{ e^{cos(t)}} = frac{sin(t)}{ e^{sin(t)}} $$
                      Let $ h(t) = t e^{-t} $. Then the upper equation becomes $h(cos(t)) = h(sin(t))$ . As $ h $ is monotonic increasing(as $h'ge 0$ on [0,1]) the equation will only be true where $cos(t) = sin(t)$. So the equation will only be true for $t = pi/4, text{ or } 5pi/4 $, or $sin(t) =cos(t) = pm sqrt{2}/2 $. So Max will be attained as $2e^{sqrt{2}/2}$ and min as $2e^{-sqrt{2}/2}$






                      share|cite|improve this answer











                      $endgroup$









                      • 1




                        $begingroup$
                        It is not true for $t = - pi/4 $. $sin(- pi/4)={- sqrt{2}/2}$ and $cos(- pi/4)={sqrt{2}/2}$
                        $endgroup$
                        – Piotr Wasilewicz
                        Feb 1 at 10:02








                      • 1




                        $begingroup$
                        Good catch! Corrected.
                        $endgroup$
                        – Maksim
                        Feb 1 at 10:30






                      • 1




                        $begingroup$
                        I think you mean $5pi/4$ :)
                        $endgroup$
                        – Piotr Wasilewicz
                        Feb 1 at 11:21










                      • $begingroup$
                        You are so right :-)
                        $endgroup$
                        – Maksim
                        Feb 1 at 11:53














                      1












                      1








                      1





                      $begingroup$

                      No need to use Lagrange at all. Substitute $x$ and $y$ by $sin(t)$ and $cos(t)$ the problem becomes maximizing $g(t):=( e^{sin(t)} + e^{cos(t)} )$. Then $g'(t) = e^{sin(t)}cos(t) - e^{cos(t)} sin(t) $. Then $g'(t)$ then is zero iff
                      $$ frac{cos(t)}{ e^{cos(t)}} = frac{sin(t)}{ e^{sin(t)}} $$
                      Let $ h(t) = t e^{-t} $. Then the upper equation becomes $h(cos(t)) = h(sin(t))$ . As $ h $ is monotonic increasing(as $h'ge 0$ on [0,1]) the equation will only be true where $cos(t) = sin(t)$. So the equation will only be true for $t = pi/4, text{ or } 5pi/4 $, or $sin(t) =cos(t) = pm sqrt{2}/2 $. So Max will be attained as $2e^{sqrt{2}/2}$ and min as $2e^{-sqrt{2}/2}$






                      share|cite|improve this answer











                      $endgroup$



                      No need to use Lagrange at all. Substitute $x$ and $y$ by $sin(t)$ and $cos(t)$ the problem becomes maximizing $g(t):=( e^{sin(t)} + e^{cos(t)} )$. Then $g'(t) = e^{sin(t)}cos(t) - e^{cos(t)} sin(t) $. Then $g'(t)$ then is zero iff
                      $$ frac{cos(t)}{ e^{cos(t)}} = frac{sin(t)}{ e^{sin(t)}} $$
                      Let $ h(t) = t e^{-t} $. Then the upper equation becomes $h(cos(t)) = h(sin(t))$ . As $ h $ is monotonic increasing(as $h'ge 0$ on [0,1]) the equation will only be true where $cos(t) = sin(t)$. So the equation will only be true for $t = pi/4, text{ or } 5pi/4 $, or $sin(t) =cos(t) = pm sqrt{2}/2 $. So Max will be attained as $2e^{sqrt{2}/2}$ and min as $2e^{-sqrt{2}/2}$







                      share|cite|improve this answer














                      share|cite|improve this answer



                      share|cite|improve this answer








                      edited Feb 1 at 11:53

























                      answered Jan 31 at 10:24









                      MaksimMaksim

                      1,00719




                      1,00719








                      • 1




                        $begingroup$
                        It is not true for $t = - pi/4 $. $sin(- pi/4)={- sqrt{2}/2}$ and $cos(- pi/4)={sqrt{2}/2}$
                        $endgroup$
                        – Piotr Wasilewicz
                        Feb 1 at 10:02








                      • 1




                        $begingroup$
                        Good catch! Corrected.
                        $endgroup$
                        – Maksim
                        Feb 1 at 10:30






                      • 1




                        $begingroup$
                        I think you mean $5pi/4$ :)
                        $endgroup$
                        – Piotr Wasilewicz
                        Feb 1 at 11:21










                      • $begingroup$
                        You are so right :-)
                        $endgroup$
                        – Maksim
                        Feb 1 at 11:53














                      • 1




                        $begingroup$
                        It is not true for $t = - pi/4 $. $sin(- pi/4)={- sqrt{2}/2}$ and $cos(- pi/4)={sqrt{2}/2}$
                        $endgroup$
                        – Piotr Wasilewicz
                        Feb 1 at 10:02








                      • 1




                        $begingroup$
                        Good catch! Corrected.
                        $endgroup$
                        – Maksim
                        Feb 1 at 10:30






                      • 1




                        $begingroup$
                        I think you mean $5pi/4$ :)
                        $endgroup$
                        – Piotr Wasilewicz
                        Feb 1 at 11:21










                      • $begingroup$
                        You are so right :-)
                        $endgroup$
                        – Maksim
                        Feb 1 at 11:53








                      1




                      1




                      $begingroup$
                      It is not true for $t = - pi/4 $. $sin(- pi/4)={- sqrt{2}/2}$ and $cos(- pi/4)={sqrt{2}/2}$
                      $endgroup$
                      – Piotr Wasilewicz
                      Feb 1 at 10:02






                      $begingroup$
                      It is not true for $t = - pi/4 $. $sin(- pi/4)={- sqrt{2}/2}$ and $cos(- pi/4)={sqrt{2}/2}$
                      $endgroup$
                      – Piotr Wasilewicz
                      Feb 1 at 10:02






                      1




                      1




                      $begingroup$
                      Good catch! Corrected.
                      $endgroup$
                      – Maksim
                      Feb 1 at 10:30




                      $begingroup$
                      Good catch! Corrected.
                      $endgroup$
                      – Maksim
                      Feb 1 at 10:30




                      1




                      1




                      $begingroup$
                      I think you mean $5pi/4$ :)
                      $endgroup$
                      – Piotr Wasilewicz
                      Feb 1 at 11:21




                      $begingroup$
                      I think you mean $5pi/4$ :)
                      $endgroup$
                      – Piotr Wasilewicz
                      Feb 1 at 11:21












                      $begingroup$
                      You are so right :-)
                      $endgroup$
                      – Maksim
                      Feb 1 at 11:53




                      $begingroup$
                      You are so right :-)
                      $endgroup$
                      – Maksim
                      Feb 1 at 11:53


















                      draft saved

                      draft discarded




















































                      Thanks for contributing an answer to Mathematics Stack Exchange!


                      • Please be sure to answer the question. Provide details and share your research!

                      But avoid



                      • Asking for help, clarification, or responding to other answers.

                      • Making statements based on opinion; back them up with references or personal experience.


                      Use MathJax to format equations. MathJax reference.


                      To learn more, see our tips on writing great answers.




                      draft saved


                      draft discarded














                      StackExchange.ready(
                      function () {
                      StackExchange.openid.initPostLogin('.new-post-login', 'https%3a%2f%2fmath.stackexchange.com%2fquestions%2f3094682%2fmax-and-min-of-fx-y-exey-on-x2y2-1%23new-answer', 'question_page');
                      }
                      );

                      Post as a guest















                      Required, but never shown





















































                      Required, but never shown














                      Required, but never shown












                      Required, but never shown







                      Required, but never shown

































                      Required, but never shown














                      Required, but never shown












                      Required, but never shown







                      Required, but never shown







                      Popular posts from this blog

                      Can a sorcerer learn a 5th-level spell early by creating spell slots using the Font of Magic feature?

                      ts Property 'filter' does not exist on type '{}'

                      Notepad++ export/extract a list of installed plugins